What is the nth term rule of -3,-10,-17,-24,-31?

1 Answer

Answer :

Subtract 7 from each number, so the 9th number would be 59.

Related questions

Description : What is the nth term of the sequence 3 10 17 24?

Last Answer : 7n - 4

Description : The nth term -4,-1,4,11,20,31?

Last Answer : 14112027

Description : What is the nth term for 15,7,-1,-9,-17?

Last Answer : 7n+4

Description : What is the nth term for 15,7,-1,-9,-17?

Last Answer : 7n+4

Description : What is the nth term rule of the quadratic sequence 7 14 23 34 47 62 79 . . .?

Last Answer : It is T(n) = n2 + 4*n + 2.

Description : What is the nth term rule of the quadratic sequence and- 7 and -6 and - 3 2 9 18 29 .?

Last Answer : It is T(n) = n2 - 2n + 6

Description : What is the nth term rule if the linear sequence 1,7,13,19,25?

Last Answer : 6n-5 is the nth term of this sequence

Description : What is the nth term rule of the quadratic sequence below 4 , 15 , 30 , 49 , 72 , 99 , 130 , . . .?

Last Answer : fsedaz sd

Description : What is the nth term rule of the quadratic sequence below 4 , 15 , 30 , 49 , 72 , 99 , 130 , . . .?

Last Answer : fsedaz sd

Description : 1, 2, 3, 4, 5, 6, 7, 8, 9, 10, 11, 12, 13, 14, 15, 16, 17, 18, 19, 20, 21, 23, 24, 25, 26, 27, 28, 29, 30, 31, 32, 33, 34, 35, 36, 37, 38, 39, 40, 41, 42, 43, 44, 45, 46, 47, 48, 49, 50'. What number is missing? -Riddles

Last Answer : 22

Description : lf the series 4,5,8,13,14,17,22,........ is continued in the same pattern, which one of the following is not a term of this series? (A) 31 (B) 32 (C) 33 (D) 35

Last Answer : Answer: C

Description : What is the Nth term of 4 7 10 13...?

Last Answer : The nth term is: 3n+1 and so the next number will be 16

Description : What is the nth term for 22.18.14.10.6?

Last Answer : 4n-2

Description : What is nth term for 5 10 20 40 80?

Last Answer : Need answer

Description : The ages of Gaurav and Goutham will be in the ratio 5 : 7 after ten years from now and will be in the ratio 13 : 18 after twelve years from now. Find the ratio of the sum of their ages 10 years hence to the sum of their ages 12 ... hence. 1 : 25 : 27 2 : 13 : 15 3 : 14 : 17 4 : 18 : 23 5 : 30 : 31

Last Answer : 5 : 30 : 31

Description : Write a difinition for the nth term of senquence A?

Last Answer : please help

Description : What is the nth term for -2 -4 -6?

Last Answer : Need answer

Description : What is the nth term for 2 5 8 11?

Last Answer : Need answer

Description : What is the nth term for 12 13 14 15?

Last Answer : Need answer

Description : What is the nth term for 20 16 12 8?

Last Answer : Need answer

Description : What is the nth term for 8 16 26 38 52?

Last Answer : Each number is increasing by increments of 8 10 12 14 ... etcand so the next number will be 52+16 = 68

Description : What is the first 5 terms of the sequence with nth term 6n-1?

Last Answer : 5, 11, 17, 23, 29

Description : What is the nth term of the arithmetic sequence 22 15 8 1 ...?

Last Answer : The nth term is -7n+29 and so the next term will be -6

Description : What is the nth term of -7 -3 1 5?

Last Answer : Without further restrictions it can be any of an infinite number of formulae.For example, U{n} = (11n⁴ - 110x³ + 385x² - 518x + 176)/8 which gives the next term as 42.However, if it is an ARITHMETIC SEQUENCE (as I suspect your ... -3) - (-7) = 4→ 0th term is -7 - 4 = -11→ nth term U{n} = 4n - 11

Description : What is the nth term of 1 8 15 22 29?

Last Answer : What is the 100th 1,8,15,22

Description : What is the nth term of -1 5 15 29 47 69?

Last Answer : What is the answer ?

Description : How do you find the nth term for the sequence 3 9 27 81 243?

Last Answer : According to Wittgenstein's Finite Rule Paradox every finite sequence of numbers can be a described in infinitely many ways - some simple, some complicated but all equally valid. One possible rule isT(n) = (2*n4 - 16*n3 + 52*n2 - 68*n + ... n = 1, 2, 3, ...Another is U(n) = 3n for n = 1, 2, 3, ...

Description : What is the nth term of 04163664?

Last Answer : It is not possible to find the nth term from the giveninformation.

Description : What is the nth term formula for the sequence 4 9 16 25 36?

Last Answer : (n+1)^2 Please tell me you know what that means.

Description : What is the nth term to -6 -3 0 3 6?

Last Answer : n-9+3

Description : What is the nth term of 4 1 -2 -5 -8?

Last Answer : Need answer

Description : What is the nth term of 1 8 15 22 29?

Last Answer : What is the 100th 1,8,15,22

Description : What is the nth term of 2,4,6,8?

Last Answer : 2000

Description : What are the first five terms of the sequence with nth term is 7n + 3?

Last Answer : 10

Description : The expression for the nth term of a sequence is 4n + 6. What are the first three terms in the sequence?

Last Answer : 10

Description : What is the nth term of -2 -8 -18 -32 - 50?

Last Answer : The nth term is 2n2. (One way to find that is to notice at all the numbers are even, then divide them by 2. The sequence becomes 1, 4, 9, 16, 25, which are the square numbers in order.)

Description : A sequence has nth term formula 3n+5. What is the fourth term?

Last Answer : ?

Description : When is National Panchayati Raj day celebrated? a. 10 th April b. 24 th April c. 31 st March d. 1 st May

Last Answer : b. 24 th April

Description : Given the following data pairs (x, y), find the regression equation. (1, 1.24), (2, 5.23), (3, 7.24), (4, 7.60), (5, 9.97), (6, 14.31), (7, 13.99), (8, 14.88), (9, 18.04), (10, 20.70) a. y = 0.490 x - 0.053 b. y = 2.04 x c. y = 1.98 x + 0.436 d. y = 0.49 x

Last Answer : c. y = 1.98 x + 0.436

Description : Which Article of Indian Constitution is related with the Protection of the interests of the minorities? (1) Article 17 (2) Article 29 (3) Article 30 (4) Article 31

Last Answer : (3) Article 30 Explanation: Article 30 is about right of minorities to establish and administer educational institutions. In article 31 compulsory acquisition of property is mentioned.

Description : When a voltage of 110 VDC is applied to the circuit illustrated with a resistance of 12 ohms the current will be________. EL-0018 A. .11 amps B. 1.31 amps C. 9.17 amps D. 122m amps

Last Answer : Answer: C

Description : Given the following specification, which of the following values for age are in the SAME equivalence partition? If you are less than 18, you are too young to be insured. Between 18 and 30 inclusive, you will receive a 20% discount. Anyone ... 17, 18, 19. B. 29, 30, 31. C. 18, 29, 30. D. 17, 29, 31.

Last Answer : C. 18, 29, 30.

Description : Insert the missing number: 16/32, 15 /33, 17/31, 14/34, ? (A) 19/35 (B) 19/30 (C) 18/35 (D) 18/30

Last Answer : Answer: D 16/32 then 15/33 means -1/+1, 17/31 means +2/-2, 14/34 means -3/+3 next +4/-4 i.e 18/30

Description : In the following question, select the odd number pair from the given alternatives.  Options:  1) 11-13  2) 17-19 3) 23-31  4) 31-37 

Last Answer : Correct Answer: 23 - 31 

Description : What is the rule for the number pattern 1 6 16 31?

Last Answer : The rule is 5, 10, 15 and so the next number will be 20+31 =51

Description : Is 0.24 greater than or less than 0.31?

Last Answer : Less than.

Description : How many times does 24 go into 31?

Last Answer : 3168 divided by 24

Description : How many times does 24 go into 31?

Last Answer : 3168 divided by 24

Description : 31.24 to 1 decimal place?

Last Answer : 5

Description : Beat instruction register prepared in form a) MS 32 b) MS 31 c) MS 30 d) MS 24

Last Answer : a) MS 32